Monday, December 4, 2017

Chapter 5 Solutions

These are explanations that I had emailed to various students while we were working on this chapter. Use this information as you review old problems, or while you're studying for the final exam.
Chp 5 Review "T" Answers:

Chp 5 Review (pg. 241): 

R9ai) Make the exponent "one bigger", then divide by the new exponent.  So, your integrated equation (which is the anti-derivative) is:

-x^-1

Then, plug in 5, then plug in 1, then subtract:

(-5^-1) - (-1^-1)

Remember when you're figuring these out: a negative exponent can be changed to a positive by using the reciprocal.

(-1/5) - (-1/1)

-1/5 + 1

4/5

R9aii) Because there's an "inside" function, and there appears to be another function multiplied, you could use U-substitution.  Make u equal to the inside function:

u = x^2 + 3

Then find du by finding the derivative of u:

du = 2x dx

Your given integral only has x dx, so divide both sides of the du equation by 2 to make it look the same:

1/2 du = x dx

Now substitute back into the integral.  You should have only u (no x).

1/2 (integral sign) u^5 du

Now use the power rule to integrate (make the exponent one bigger, then divide by the new exponent).

1/2 * 1/6 u^6

1/12 u^6

Substitute u back out:

1/12(x^2 + 3)^6

Plug in 4, then plug in 3, then subtract:

1/12(4^2 + 3)^6 - 1/12(3^2 + 3)^6

1/12(19)^6 - 1/12(12)^6

= 3,671,658.083

R9aiii) Integrate each term separately.  The derivative of what gives you sin x? -->  -cos x

The derivative of what gives you 5? -->  5x

So, your integrated equation is:

(integral sign) -cos x - 5x

Plug in pi, then plug in 0, then subtract:

(-cos pi - 5pi) - (-cos 0 - 5*0)

(1 - 5pi) - (-1)

2 - 5pi

R9d) I found a picture online that illustrates the statement in this problem (see below).  It's basically saying that if your entire integral goes from a to b, that you could split that whole area at some "c" location.  This would make it two areas, where one goes from a to c, and the other goes from c to b.

Picture from:
http://www.drcruzan.com/Images/Mathematics/DefiniteIntegrals/DecompositionGraph.png

R10a) Check the answer in the back of the book for this picture.

R10b) This says that displacement (dy) is equal to the velocity (v) times the time (t). Write an equation saying this:

dy = v * dt

To “write dy in terms of t and dt”, substitute the velocity equation they gave you in for “v”.

dy = 150t^0.5 * dt

R10c) Remember how displacement, velocity, and acceleration are related to each other:

displacement —derivative—> velocity —derivative—> acceleration

acceleration —integral—> velocity —integral—> displacement

So in this problem, if they want you to use an integral to find displacement, you’ll need to integrate the velocity equation. The integration method they’re asking you to use here is a Riemann Sum. The answer in the back of the book is a different form than the way we’ve been writing them in class, but you could write a Riemann Sum however you’d like. Here’s one example:

Mid-Riemann Sum of 3 rectangles. You’re integrating velocity, so you’ll use the velocity equation to find the height of each rectangle. The width of each rectangle would be 3:

3 * v(1.5) + 3 * v(4.5) + 3 * v(7.5)

…or could be written as:

3 * ( v(1.5) + v(4.5) + v(7.5) )

R10d) The limit of ANY Riemann Sum as dt (the width of each rectangle) approaches zero will be equal to the exact definite integral. This is because as the width of the rectangles approaches zero, there must be more and more rectangles being used.

So, to find this limit, you could just find the integral by using the Fundamental Theorem of Calculus (FTC).

(integral sign from 0 to 9) 150t^0.5

Make the exponent “one bigger”, then divide by the new exponent:

100t^1.5 (bounded 0 to 9) =

100(9)^1.5 - 100(0)^1.5 =

2700 feet

R10e) This problem is trying to help you see that two smaller integrals added together will equal the larger integral. Find the integral from 0 to 4, and find the integral from 4 to 9. It should equal the same answer you got in Part D.

Sec 5.10 worksheet

These links are Educreations videos I made for Sec 5.10 homework (worksheet).  Make sure your sound is on!

Sec 5.10 worksheet #1 (this is an explanation of the problem we already did in class)
Sec 5.10 homework #1 & #3

Sec 5.9 help: Fundamental Theorem of Calculus

This link is an Educreations video I made for the Sec 5.9 homework.

Examples of how to calculate using FTC (using problems from Sec 5.9 in the book)

Winter Break Packet:

2c) An easy way to do this is to plug in a number really close to x = 2. I would try x = 1.99 and x = 2.01 (do both, just to make sure that you’re getting the same answer on both sides of the limit).

You could try factoring the numerator and denominator as the “difference of two squares”, but I don’t think it will help too much. The numerator becomes: (2 + x)(2 - x). The denominator could be factored the same way (like using the square root of 3 in each factor, etc.), but it wouldn’t create anything to cancel out factors in the numerator.

2d) The “trick” here is to recognize that this problem is actually the forward difference quotient:

lim        f(x + h) - f(x)
h—>0           h

See how the two functions in the numerator are the same, but the first one has a “+h" in it? And the denominator is just an “h”, and the limit is h—>0.

So, the original function that the forward difference quotient is being applied to is: 3 / (x - 2)

This means that this question is just asking you to find the derivative of f(x) = 3 / (x - 2). You could use the quotient rule to do this, or you could roll the denominator up and then use the chain rule:

f(x) = 3(x - 2)^-1

f’(x) = -3(x - 2)^-2

f’(x) = -3 / (x - 2)^2

3f) I haven’t taught you this yet! Stay tuned for class tomorrow.

3g) An integral is an anti-derivative, so we need to find the anti-derivative of f(x) = x^2 + 1. Remember, “add 1 to the exponent, then divide by the new exponent”.

g(x) = 1/3x^3 + x + c

3h) This tells you that when x = 3, the g(x) equation equals 1. Plug these in to find c:

1 = 1/3(3)^3 + 3 + c
1 = 1/3(27) + 3 + c
1 = 9 + 3 + c
1 = 12 + c
-11 = c

g(x) = 1/3x^3 + x - 11

6) Factor the numerator as the difference of two squares. It’s like doing this:

x^2 - 25 = (x + 5)(x - 5)

But here, it’s like this:

x - 1 = (sqrt(x) + 1)(sqrt(x) - 1)

Now your function looks like:

(sqrt(x) + 1)(sqrt(x) - 1) / (sqrt(x) - 1)

You can cancel out a factor from the top and bottom, which will then let you plug in x = 1 to find the limit. Here’s your equation after canceling:

lim        sqrt(x) + 1
x—>1

Then plug in x = 1:

sqrt(1) + 1
1 + 1
= 2


17) Finding the area between a curve and the x-axis is the same as finding its integral. Here, you need to find the points where the function intersects the x-axis so you'll know which bounds to use in your integral.

y = x^2 - x - 6

0 = x^2 - x - 6

0 = (x - 3)(x + 2)

x = 3 and -2

So, you need to integrate from -2 to 3:

(integral sign -2 to 3) x^2 - x - 6 * dx

= 1/3x^3 - 1/2x^2 - 6x

= [ 1/3(3)^3 - 1/2(3)^2 - 6(3) ] - 1/3(-2)^3 - 1/2(-2)^2 - 6(-2) ]

= [ 9 - 4.5 - 18 ] - [ -8/3 - 2 + 12 ]

= -13.5 - 22/3

= -20.833

Because the question is asking us about the area under the curve, we'll answer in terms of the positive value, 20.833.

18a) This question is basically giving you the answer, and asking you to show how you find it. Find the implicit derivative of both sides of the equation to answer this question.

x^2 - sin y = y + 4

Remember that every time you find a derivative involving "y", you need to multiply by dy/dx Derivative is:

2x - cos y * dy/dx = dy/dx

Get all the dy/dx terms on one side, and all the other terms on the other side:

2x = dy/dx + cos y * dy/dx

Factor the "dy/dx" out of both terms, then divide to get the "dy/dx" to find the answer.

2x = dy/dx(1 + cos y)

2x / (1 + cos y) = dy/dx

18b) To find a tangent line equation, you need a slope and a point.

Find the point at y = 0 by plugging in y = 0 to the original equation:

x^2 - sin 0 = 0 + 4
x^2 = 4
x = 2 and -2

Find the slope at both (x,y) points by plugging in (2,0) and (-2,0) to the dy/dx equation:

dy/dx = 2(2) / (1 + cos 0) = 4 / 2 = 2

dy/dx = 2(-2) / (1 + cos 0) = -4 / 2 = -2

You can write two tangent line equations. One has a point at (2,0) and dy/dx = 2. The other has a point at (-2,0) and dy/dx = -2.

Answers:

y - 0 = 2(x - 2)
y = 2x - 4

y - 0 = -2(x + 2)
y = -2x - 4

AP Q #6) WHOOPS! We haven't learned this one yet. Skip it!

AP Q #7) Start by finding the tangent line equation. You need a slope and a point. They gave you the point, but you need to find the derivative equation to find the slope.

Find the derivative f'(x) by using the quotient rule:

f(x) = sec x / (x + 2)

f'(x) = [sec x tan x * (x + 2) - sec x] / (x + 2)^2

f'(x) = sec x * [tan x * (x + 2) - 1] / (x + 2)^2

f'(x) = [tan x * (x + 2) - 1] / [cos x * (x + 2)^2]

Plug in x = 0 to find f'(0):

f'(0) = [tan 0 * (0 + 2) - 1] / [cos 0 * (0 + 2)^2]

f'(0) = [0 * (2) - 1] / [1 * 4]

f'(0) = -1/4

Your tangent line equation is:

y - 1/2 = -1/4(x - 0)

y = -1/4x + 1/2

Now use this to approximate f(0.1) by plugging in x = 0.1 for x in the tangent line equation:

y = -1/4(0.1) + 1/2 = -0.025 + 0.5 = 0.475

Sec 5.6 (pg. 208):

General help: Mean Value Theorem
These links are Educreations videos I made for Sec 5.6 Exploration Wkst. last year. They should mostly line up with the worksheet we did in class this year.
Sec 5.6 Exploration Problem #1
Sec 5.6 Exploration Problem #2
Sec 5.6 Exploration Problem #3
Sec 5.6 Exploration Problem #4

Sec 5.6 Exploration - BACK OF WORKSHEET (lots of important information about Mean Value Theorem in this video especially)

3) The Mean Value Theorem (MVT) says that if a graph is differentiable, then there is some point in the interval that has the same slope as the average slope between the two endpoints.


So…

This function, 6/x, isn't continuous and differentiable everywhere, but it is for the interval we're looking at: [1, 4].  So, the hypotheses are met!

1st step: Find the average slope between the endpoints of the interval:

g(4) - g(1) / 4 - 1

3/2 - 6 / 3

-4.5 / 3

-3/2

2nd step: Set the derivative equal to the average slope.  (Reason: We are trying to find the x = c point where the slope is the same as the average slope).

g'(x) = -6/x^2

-6/x^2 = -3/2

3x^2 = 12

x^2 = 4

Final answer: x = 2

4) Same ideas!

1st step: Find the average slope between the endpoints.

f(2) - f(-1) / 2 - -1

16 - 1 / 3

15 / 3

5

2nd step: Set the derivative equal to the average slope.

f'(x) = 4x^3

4x^3 = 5

x^3 = 1.25

Final answer: x = 1.077

Sec 5.5: Riemann Sum

Note: This is a Khan Academy/YouTube video that I didn't create, but it's a good explanation. 
How do I find a "Lower Riemann Sum"?

Sec 5.4 (pg. 193):

33) You can't find the integral (anti-derivative) the way it's written in the book, because there isn't a "derivative of the inside" anywhere in the integral, which would have shown up from doing the chain rule.

Instead, you need to multiply it out, then find the integral by doing reverse power rule.

So…

(x^2 + 5)^3 multiplied out becomes…

[ Remember Pascal's Triangle? (x + h)^3 = x^3 + 3x^2h + 3xh^2 + h^3 ]

x^6 + 15x^4 + 75x^2 + 125

Now you can find the integral!  "Make the exponent one bigger, then divide by the new exponent."

1/7x^7 + 3x^5 + 25x^3 + 125x + c

Monday, November 13, 2017

Chapter 4 Solutions

These are explanations that I had emailed to various students while we were working on this chapter. Use this information as you review old problems, or while you're studying for the final exam.

Educreations Video:

YouTube Video:

Chapter 4 Review Packet:

These links are Educreations videos I made. Make sure your sound is on! (I might refer to the book problem number instead of the packet problem number in the video, since I made the videos last year).

13) You're given y = tan^-1 (4x^2). Start by moving "tan" to the other side:

tan y = 4x^2

Next, set up a right triangle where "y" is an angle. From SOH CAH TOA, we know tan = opp/adj. Here, we have tan y = 4x^2 / 1. So, your triangle should have a side OPPOSITE of y that says 4x^2, and ADJACENT to y that says 1.

To find the missing hypotenuse side, use the Pythagorean Theorem:

(4x^2)^2 + 1^2 = c^2
16x^4 + 1 = c^2
sqrt(16x^4 + 1) = c

Back to the starting equation: tan y = 4x^2

The actual goal is to find the derivative of this equation. Use implicit differentiation:

d/dx ( tan y = 4x^2 )

sec^2 y * dy/dx = 8x ... then get dy/dx by itself:

dy/dx = 8x / sec^2 y

dy/dx = 8x * cos^2 y

We can use that right triangle to figure out what cos y is equal to, because "y" was an angle in our triangle. We know cos = adj/hyp. In our triangle the ADJACENT was 1 and the HYPOTENUSE was sqrt(16x^4 + 1).

cos y = 1 / sqrt(16x^4 + 1)

Then... dy/dx = 8x * cos^2 y ...becomes...

dy/dx = 8x * (1 / sqrt(16x^4 + 1))^2

dy/dx = 8x * (1 / (16x^4 + 1))

dy/dx = 8x / (16x^4 + 1)

24f) Because you have both x and y on the same side, you need to do an implicit derivative. Remember, every time you find a derivative with y in it, you need to multiply by dy/dx (or y’ for short).

We’re starting with:

5x^3*y^7 = y^1.6

Let’s find the derivative on the right of the = sign first. Just do the power rule, but multiply by y’:

        = 1.6y^0.6 * y’

To find the derivative on the left side, we need to use product rule, because we have two functions multiplied. It will help to write them out as “u” and “v”, then find u’ and v’.

u = 5x^3

u’ = 15x^2

v = y^7

v’ = 7y^6 * y’

Now use product rule, where the derivative will equal u’v + uv’.

15x^2 * y^7 + 5x^3 * 7y^6 * y’ = 1.6y^0.6 * y’

Now get all terms with y’ on one side, and all terms without y’ on the other side. I’ve bolded the terms with y’ and underlined the terms without y’.

15x^2 * y^7 + 5x^3 * 7y^6 * y’ 1.6y^0.6 * y’

You can subtract to move the 5x^3 * 7y^6 * y’ to the other side.

15x^2 * y^7  = 1.6y^0.6 * y’ - 5x^3 * 7y^6 * y’

Now factor out the y’, because it’s common to both terms on the right side.

15x^2 * y^7  = y’(1.6y^0.6 - 5x^3 * 7y^6)

Now divide to get y’ by itself. This is your final answer.

(15x^2 * y^7) / (1.6y^0.6 - 5x^3 * 7y^6) = y’ 

25) My answer key is actually a little shorter than it should be for this problem. Because you’re finding the derivative of an inverse trig function, this is one of those problems where you need to set up a SOH CAH TOA triangle and do an implicit derivative. Here are the steps:

First, change the equation so it’s not an inverse trig function any more:

y = 4 sin^-1 (5x^3)

y/4 = sin^-1 (5x^3)

sin (y/4) = 5x^3

Now, do two things with this equation. One, find the implicit derivative. Two, set up a SOH CAH TOA triangle with it.

Implicit derivative (remember to multiply by dy/dx any time you find a derivative involving y):

cos (y/4) * 1/4 * dy/dx = 15x^2

Then solve for dy/dx:

cos (y/4) * dy/dx = 60x^2

dy/dx = 60x^2 / cos (y/4)

SOH CAH TOA triangle:

Use your equation from before taking the implicit derivative: sin (y/4) = 5x^3. It’s always sin of the angle, so the angle must be y/4. Then, sin is equal to opp / hyp, so the opposite side must be 5x^3 and the hypotenuse must be 1. I’ll do my best to type out a triangle:

                    ----
        1    ----      |
          ----          |   5x^3
    y/4__________

Use Pythagorean Theorem to find the missing side (adj side):

A^2 + (5x^3)^2 = 1^2

A^2 + 25x^6 = 1

A^2 = 1 - 25x^6

A = sqrt(1 - 25x^6)
                    ----
        1    ----      |
          ----          |   5x^3
    y/4__________

        sqrt(1 - 25x^6)

The reason we’re doing this is because our derivative equation left off as: dy/dx = 60x^2 / cos (y/4) but we really don’t want any “y”s to show up in it. So, we’ll use this triangle to figure out what cos (y/4) is equal to. Remember, cos = adj / hyp, so…

cos (y/4) = sqrt(1 - 25x^6) / 1

cos (y/4) = sqrt(1 - 25x^6)

Back to our derivative! Now we can plug this in for cos (y/4):

dy/dx = 60x^2 / cos (y/4)

dy/dx = 60x^2 / sqrt(1 - 25x^6)

26) This is probably the toughest problem on the entire assignment. I strongly encourage you to look at the work that’s written on my answer key in the Google Drive folder. But, to help you decode all those steps, here are the big steps you take along the way:

Step 1) You’re given an equation with x and y on the same side, so you need to use implicit differentiation. Remember to multiply by dy/dx every time you find a derivative involving y.

After you find the derivative, get all terms with dy/dx on one side, and all other terms on the other side. Then, factor out the dy/dx. Then, get dy/dx by itself by dividing.

Step 2) Next, you need to use the points “where x = -2”. But, they only told you that x = -2, and they didn’t tell you what y is equal to at that point. Use your ORIGINAL EQUATION to find the necessary y-values. Plug in x = -2 everywhere x shows up, then solve for y (you’ll need to use quadratic formula to solve for y at some point in this work).

Step 3) To actually find the slope at x = -2, now you’ll use the dy/dx equation (because the derivative equation will tell you the slope at any point), and you’ll plug in the (x, y) values you found earlier.

Sec 4.8:

These links are Educreations videos I made for Implicit Derivatives.  Make sure your sound is on!


Sec 4.5:

These links are Educreations videos I made for Derivatives of Inverses.  Make sure your sound is on!

13) There are two main ways to do this problem: Derivative of y = sin^-1(4x)

Short way, but requires memorization:

According to the table on pg. 150, the derivative of y = sin^-1(x) is

1 / sqrt(1 - x^2)

Apply the chain rule to this problem! The derivative of the "outside", which is sin^-1, will be the formula given above. Keep the "inside", which is 4x, the same. Then multiply by the derivative of the inside.

1 / sqrt(1 - (4x)^2) * 4

= 4 / sqrt(1 - 16x^2)

Long way, but you don't have to memorize anything:

Start by finding "sin" of each side of the equation:

y = sin^-1(4x)

sin(y) = sin(sin^-1(4x))

sin(y) = 4x

Next, find the implicit derivative of each side of the equation:

cos(y) * dy/dx = 4

dy/dx = 4 / cos(y)

dy/dx = 4 * sec(y)

We can't leave our answer in terms of "y", so we will draw a right triangle using the initial information to figure out what "sec(y)" is equal to.

Initial information: sin(y) = 4x

sin = opp / hyp, so draw a triangle with opposite side = 4x and hypotenuse = 1.

Use Pythagorean Theorem to find missing side of triangle:

(4x)^2 + b^2 = 1^2

16x^2 + b^2 = 1

b^2 = 1 - 16x^2

b = sqrt(1 - 16x^2)

When you're solving with SOH CAH TOA, we know that cos = adj / hyp. Because sec is the reciprocal of cos, then sec must be equal to the reciprocal of the cos proportion.

sec = hyp / adj

sec(y) = 1 / sqrt(1 - 16x^2)

Substitute this back into the derivative equation you found, dy/dx:

dy/dx = 4 * sec(y)

dy/dx = 4 * 1 / sqrt(1 - 16x^2)

dy/dx = 4 / sqrt(1 - 16x^2)

23) You need to do product rule with the first part of the equation, then chain rule with the second part.  Here's what each piece would look like when you find the derivative, then simplify:

x * sin^-1 x

When you find the derivative:

1 * sin^-1 x + x * 1/sqrt(1-x^2)

The derivative of sin^-1 x is given on page 150 in the textbook.  We can simplify this equation to:

sin^-1 x + x/sqrt(1-x^2)

For the next part, use the chain rule: (1-x^2)^1/2

1/2(1-x^2)^-1/2 * -2x

Then simplify.  The 1/2 * 2 will cancel out, leaving just -x.  We can move the parentheses to the denominator to make the exponent positive.  Then, once it's a positive 1/2 exponent, we can rewrite it as a square root.  Simplified version:

-x/sqrt(1-x^2)

So, here's what we have for the derivative of v:

v' = sin^-1 x + x/sqrt(1-x^2) - x/sqrt(1-x^2)

Notice that the last two terms are the same, except one is positive x and one is negative x, so they will cancel.  We are left with:

v' = sin^-1 x

25a) tan (theta) = opp/adj, so tan (theta) = x/100. You can then solve for (theta) by taking the inverse tan of both sides to get:

(theta) = tan^-1 (x/100)

25b) To find d(theta)/dx, you need to find the derivative of the equation from Part "a" of this problem.

(theta) = tan^-1 (x/100)

Find the derivative with respect to x on each side. This problem will be easier to just use the FORMULA for derivative of tan^-1. Be sure to apply chain rule, because you have an outside function (tan^-1) and an inside function (x/100):

d(theta)/dx = 1 / (1 + (x/100)^2) ... then also multiplied by 1/100, which is the derivative of x/100.

d(theta)/dx = 1 / (1 + (x/100)^2) * 1/100

We can algebraically simplify this equation a lot! Start by squaring the x/100:

d(theta)/dx = 1 / (1 + x^2/10000) * 1/100

Then, because you have two fractions multiplied together, we can multiply the top and then the bottom:

d(theta)/dx = 1 / (100 + 100x^2/10000)

d(theta)/dx = 1 / (100 + x^2/100)

It's awkward to have a fraction within the denominator of another fraction, so we should find a way to combine the two terms in the denominator. For just these two terms, find a common denominator, then add them together:

d(theta)/dx = 1 / (10000/100 + x^2/100)

d(theta)/dx = 1 / ((10000+ x^2)/100)

Then, instead of having this fraction in the denominator, you can multiply by its reciprocal:

d(theta)/dx = 1 * 100/(10000+ x^2)

d(theta)/dx = 100/(10000+ x^2)

WHEW! Finally, to find d(theta)/dt, let's think about our alternate definition of the chain rule, and how we could get from d(theta)/dx to d(theta)/dt:

d(theta)/dt = d(theta)/?? * ??/dt

The missing pieces..."??"...can be "dx":

d(theta)/dt = d(theta)/dx * dx/dt

So, we can use the d(theta)/dx equation we already found, and simply multiply it by dx/dt to force it to be the d(theta)/dt equation we're looking for:

d(theta)/dt = 100/(10000+ x^2) * dx/dt

25c) Plug in x = 500 and d(theta)/dt = -0/.04:

-0.04 = 100/(10000+ 500^2) * dx/dt

When this question asks you, "How fast is the truck going?", they're really just asking you what dx/dt is equal to. This is because the "x" units are feet, and the "t" units are seconds, so "dx/dt" is ft/sec.

When you solve this equation algebraically for dx/dt, you get 104, so your answer is "104 ft/sec". You can convert this to miles per hour by using pre-calc unit conversion strategies?

104 ft * 60 sec * 60 min * 1 mile    = 71 mil/hr
sec        1 min      1 hr     5280 ft

Sec 4.4:

13) For this problem, the most "outside function" is ^(5/4).
The next inside function is sec.
The most inside function is 4x.

So, take the derivative of the ^(5/4) first:

y = 8(sec 4x)^(5/4)

y' = 8 * 5/4(sec 4x)^(1/4)

Then multiply by the derivative of sec:

y' = 8 * 5/4(sec 4x)^(1/4) * sec 4x * tan 4x

Then multiply by the derivative of 4x:

y' = 8 * 5/4(sec 4x)^(1/4) * sec 4x * tan 4x * 4

This is your entire derivative!  The last step is to simplify by combining the constant numbers that are multiplied together.

8 * 5/4 * 4 = 40

Also, you have (sec 4x)^(1/4) and (sec 4x).  These can be combined, because they are both sec and they both have 4x on the inside.  Because the first one has an exponent of 1/4 and the second one has an exponent of 1, they combine to:

(sec 4x)^(5/4)

So, the equation you got when you found the derivative…

y' = 8 * 5/4(sec 4x)^(1/4) * sec 4x * tan 4x * 4

…now simplifies to this:

y' = 40 (sec 4x)^(5/4) * tan 4x

21) y = sec x csc x

This is two functions multiplied, so we need to use the product rule to find the derivative.

u = sec x
u' = sec x tan x

v = csc x
v' = -csc x cot x

So, the product rule gives us:

y' = sec x tan x * csc x + sec x * - csc x cot x

Next, try changing everything to terms of sin and cos to see if anything will cancel out.  After you cancel things out, you can change everything back to sec, csc, tan, cot.  See my work below:


24) The equation can simplify before you even find the derivative.  cot x is the same as cos x / sin x, so the cos x part will cancel out with the denominator, leaving you with 1 / sin x.  This is the same as csc x.  See below:



Then you can just find the derivative of y = csc x:

y' = - csc x cot x

Sec 4.2 (pg. 134):

General Help) This link is an Educreations video I made to teach you how to use the Product Rule. There is NO sound on this one:
Video: Product Rule Example Problems 
(There is a mistake in this video where I magically change a 20 to a 45. Sorry! I bet you can still figure out how to do the problem.)

29) ODD FUNCTION PROOF:

An odd function starts with the property f(-x) = -f(x). We will find the derivative of this equation, and will show that the end result is the same as an even function.

f(-x) = -f(x)

Find the derivative of each side, using the chain rule where appropriate:

f'(-x) * -1 = -f'(x)

Rearranged to show:

-f'(-x) = -f'(x)

Divide both sides by -1:

f'(-x) = f'(x)

This is the same as the property of an even function, so we've shown that the derivative of an odd function will give you an even function.

EVEN FUNCTION PROOF:

An even function starts with the property f(-x) = f(x). We will find the derivative of this equation, and will show that the end result is the same as an odd function.

f(-x) = f(x)

Find the derivative of each side, using chain rule where appropriate:

f'(-x) * -1 = f'(x)

Rearranged to show:

-f'(-x) = f'(x)

Divide both sides by -1:

f'(-x) = -f'(x)

This is the same as the property of an odd function, so we've shown that the derivative of an even function will give you an odd function.

35a) Start with A = L*W. When they're asking for dA/dt, they are asking you to find the derivative of your A equation with respect to t (this just means the derivative of A where t is the variable). This sounds confusing, but let's use the product rule like normal, and it should work out:

A = L*W

Product rule says: y' = u'v + uv'

The derivative of L with respect to time t will be: dL/dt
The derivative of W with respect to time t will be: dW/dt

dA/dt = dL/dt * W + L * dW/dt

35b) They're basically giving you the equations for L and W, and want you to plug them into the equation you found in 35a. Let's start by finding the derivative of L and of W:

W = 2 + 2 cos t
dW/dt = -2 sin t

L = 3 + 2 sin 2t
dL/dt = 4 cos 2t  

We actually just automatically found the derivative of W and L with respect to time t because the variable in both of these equations is t. Now let's plug all four equations back into dA/dt:

dA/dt = dL/dt * W + L * dW/dt

dA/dt = (4 cos t)*(2 + 2 cos t) + (3 + 2 sin 2t)*(-2 sin t)

Look! Without even trying, we now have the derivative of A with respect to t, because the variables in our dA equation are t.

To actually answer this book problem, we need to use dA/dt. When they ask, "At what rate is the area changing when t = 4 sec?", they are asking about the rate of change of A, or in other words the derivative of A when t = 4. So, plug t = 4 into dA/dt. Use your calculator!

dA/dt = (4 cos 4)*(2 + 2 cos 4) + (3 + 2 sin 2*4)*(-2 sin 4)
dA/dt = 7.133 ft^2 / sec

This is a positive rate of change, which means the area is INCREASING at t = 4. To figure out the units, think about the units of A (ft^2) divided by the units of t (sec). This gives you ft^2 / sec. Do the same thing for t = 5, but I'll let you figure that out on your own.

Sec 4.1 (pg. 131):

6) "Just for fun, see if you can randomly guess these answers somehow..."

The real way to do this problem is to use the Product Rule and the Quotient Rule. It's OK if you weren't able to get these answers while doing Sec 4.1 for the first time; I just wanted you guys to try it out!

p'(2) = f'(2) * g(2) + f(2) * g'(2)

q'(2) = [f'(2) * g(2) - f(2) * g'(2)] / [g(2)]^2